The Student Room Group

Pigeonhole Principal

I understand the principal, but I'm not sure how to prove it correctly, it was the one question I was really unsure of in my January exam and I think it's gonna crop up again!

"Show that if five integers are selected from the first eight positive integers, then
there always exists a pair of these five integers whose sum is 9."

So I would set up a function of X = {x_1, ... x_5},where 1<=x<=9 and Y = {{1,9}, ... {4,5}} and obviously |X|= 5 > |Y| = 4, but I'm not sure how to express this as a proper proof showing that arbitrarily f(x_j) = f(x_k)?

Thanks! :smile:
(edited 11 years ago)
Reply 1
Let X be the set of five numbers selected. If no two of the numbers summed to 9, then only one of 1 and 8 could be present in X, only one of 2 and 7 could be present in X, only one of 3 and 6 could be present in X, and only one of 4 and 5 could be present in X, so X would have at most four numbers, which is a contradiction.

I'm not sure if this uses the pigeonhole principle very well though
Reply 2
That's kinda what I blagged in the January exam and only got something like 6/9 and I'm pretty sure they want me to be more rigorous in this exam. :biggrin:
Reply 3
Original post by Brit_Miller
That's kinda what I blagged in the January exam and only got something like 6/9 and I'm pretty sure they want me to be more rigorous in this exam. :biggrin:


What I posted is a correct proof and is fully rigorous. I'm not sure if it uses the pigeonhole principle properly though. Have you looked online for a marking scheme for this paper?
Reply 4
Original post by nohomo
What I posted is a correct proof and is fully rigorous. I'm not sure if it uses the pigeonhole principle properly though. Have you looked online for a marking scheme for this paper?


I'm not saying it isn't right! I just meant I did something similar last time and got marked down, so I thought maybe there was a way you were supposed to approach the problem other than this. :smile: I've not seen a solution from the marking scheme yet, I just wondered how all of you maths whizzes would approach it.
(edited 11 years ago)
Reply 5
Original post by Brit_Miller
I'm not saying it isn't right! I just meant I did something similar last time and got marked down, so I thought maybe there was a way you were supposed to approach the problem other than this. :smile: I've not seen a solution from the marking scheme yet, I just wondered how all of you maths whizzes would approach it.


I think I have an idea of what they'd want. Consider the set

Y={(1,8),(2,7),(3,6),(4,5)}Y = \left\{ (1,8),(2,7),(3,6),(4,5)\right\}

In choosing five numbers from 1 to 8, we are choosing five numbers from the pairs in the set YY. Since there are four pairs in YY, by the Pigeonhole principle, we must choose two numbers which occur in the same pair in YY, and these two numbers sum to 99.

This proof uses the pigeonhole principle, but I don't know what mark it would get on your exam.
Reply 6
Yeah that's the structure of the proof I wanted, maybe I'm trying to be too general in the proof and should just point out the obvious!

Quick Reply

Latest